- Sun Jan 20, 2013 12:00 am
#26043
Complete Question Explanation
(The complete setup for this game can be found here: lsat/viewtopic.php?f=107&t=7219)
The correct answer choice is (B)
The stem establishes that exactly three subzones are designated for each use (H, I, R) for a total of nine designated subzones. Since no zone contains three H subzones, we can begin our analysis by separating the three H subzones into two groups (regardless of the actual development zones they are in):
Second, consider the designation of the three I subzones. Due to the I H rule, we can infer that all three I subzones must all be designated within the only zone that does not contain any H’s, an inference that eliminates answer choice (A): Third, recall that no more than 1 R is allowed in any zone containing an H subzone (third rule), and so at most 2 R’s can be distributed between the zones containing H. Therefore, at least one R must be added to the group of three I subzones: Next, we need to comply with the first rule of the game, which prohibits one of the zones (Z1) from containing any R subzones. Clearly, then, the two unassigned R subzones cannot be evenly split between the zones containing H’s: if they were, then each of the three groups would contain an R subzone in violation of the first rule. Additionally, the two remaining R’s cannot both be added to either zone containing an H, in compliance with the third rule. Accordingly, one of these R’s must be added to the group of 3 I’s and 1R, eliminating answer choice (D): Finally, let’s examine the placement of the remaining R. According to the last rule, no zone can contain an I subzone along with 3 R subzones. Therefore, the remaining R cannot be added to rightmost of the three vertical groups above, and must instead be added to one of the remaining two groups, eliminating answer choice (E): While we cannot determine exactly which development zone each block corresponds to, it is clear that the zone containing I subzones must be either Z2 or Z3. This eliminates answer choice (C), leaving us with answer choice (B), which could be true and is therefore correct.
(The complete setup for this game can be found here: lsat/viewtopic.php?f=107&t=7219)
The correct answer choice is (B)
The stem establishes that exactly three subzones are designated for each use (H, I, R) for a total of nine designated subzones. Since no zone contains three H subzones, we can begin our analysis by separating the three H subzones into two groups (regardless of the actual development zones they are in):
Second, consider the designation of the three I subzones. Due to the I H rule, we can infer that all three I subzones must all be designated within the only zone that does not contain any H’s, an inference that eliminates answer choice (A): Third, recall that no more than 1 R is allowed in any zone containing an H subzone (third rule), and so at most 2 R’s can be distributed between the zones containing H. Therefore, at least one R must be added to the group of three I subzones: Next, we need to comply with the first rule of the game, which prohibits one of the zones (Z1) from containing any R subzones. Clearly, then, the two unassigned R subzones cannot be evenly split between the zones containing H’s: if they were, then each of the three groups would contain an R subzone in violation of the first rule. Additionally, the two remaining R’s cannot both be added to either zone containing an H, in compliance with the third rule. Accordingly, one of these R’s must be added to the group of 3 I’s and 1R, eliminating answer choice (D): Finally, let’s examine the placement of the remaining R. According to the last rule, no zone can contain an I subzone along with 3 R subzones. Therefore, the remaining R cannot be added to rightmost of the three vertical groups above, and must instead be added to one of the remaining two groups, eliminating answer choice (E): While we cannot determine exactly which development zone each block corresponds to, it is clear that the zone containing I subzones must be either Z2 or Z3. This eliminates answer choice (C), leaving us with answer choice (B), which could be true and is therefore correct.
You do not have the required permissions to view the files attached to this post.